Đến nội dung

redfox nội dung

Có 96 mục bởi redfox (Tìm giới hạn từ 10-06-2020)



Sắp theo                Sắp xếp  

#691418 Chứng minh tập $\mathbb{N^*}\setminus P $ là tậ...

Đã gửi bởi redfox on 24-08-2017 - 17:52 trong Tổ hợp và rời rạc

Theo mình nghĩ ii) là $\forall q\in N,q>1,\exists c\in P,c\not\equiv 0(mod\; q)$ (nếu ii) như trên thử xét $P= \left \{ 4,6,8,...,2k,... \right \}$, và lấy $c= 4q+2$)

Để ý nếu $a,b\in P\Rightarrow ax+by\in P,\forall x,y\in N$ (theo i)). Áp dụng Sylvester ta có nếu $a,b\in P,gcd(a,b)=d\Rightarrow xd\in P,\forall x\in N,x\geq \frac{(a-d)(b-d)}{d^2}$. (*)

Gọi $d=min(gcd(a,b)|a,b\in P)$ theo (*) tồn tại $X$ sao cho $dx\in P,\forall x\in N,x\geq X$.

Giả sử $d>1$. Theo ii) lấy $p=d$ ta có tồn tại $c\in P$ không chia hết cho $d$. Đặt $d'=gcd(c,d)< d$. Lấy $x=cy+1\geq X$ ta có $xd,c\in P,gcd(xd,c)=gcd(cyd+d,c)=d'<d$ (trái với cách chọn $d$. Vậy $d=1$, theo (*) ta dễ có $N/P$ hữu hạn.

(Q.E.D)




#654503 $a_{n+1}=\frac{2}{a_{n}+a_{...

Đã gửi bởi redfox on 17-09-2016 - 17:48 trong Dãy số - Giới hạn

Đặt $b_n= a_n+\frac{1}{a_n}\Rightarrow b_{n+2}\leq \frac{b_{n+1}+b_n}{2}$ suy ra $(b_n)$ hội tụ (xét dãy phụ $M_n=max(b_n;b_{n+1})$). Đặt $limb_n=L$

Giả sử $L>2$. Chọn $0<\varepsilon <\frac{4L-8}{3L}$ và $\forall n\geq n_0,L-\varepsilon \leq b_n\leq L+\varepsilon$. Với $x_1,x_2$ là nghiệm của pt $x^2-(L-\varepsilon)x+1=0$ và $y_1,y_2$ là nghiệm của pt $y^2-(L+\varepsilon)y+1=0$ ($x_1<x_2, y_1<y_2$), ta có $a_n,\frac{1}{a_n}\in [y_1;x_1]\cup [x_2;y_2]$.

Để ý $x_1-y_1=y_2-x_2< x_2-x_1\Rightarrow \frac{z+t}{2}\notin [y_1;x_1]\cup [x_2;y_2],\forall z\in [y_1;x_1],t\in [x_2;y_2]$ mà $\frac{a_n+a_{n+1}}{2}= \frac{1}{a_{n+2}}\in [y_1;x_1]\cup [x_2;y_2]$ nên $a_n,a_{n+1}$ thuộc cùng một tập, giả sử $[y_1;x_1]$. Chứng minh tương tự ta có $a_{n+1},a_{n+2}\in [y_1;x_1]$

Nhưng $\frac{1}{a_{n+2}}=\frac{a_n+a_{n+1}}{2}\in [y_1;x_1]\Rightarrow a_{n+2}\in [\frac{1}{x_1};\frac{1}{y_1}]=[x_2;y_2]$ (mâu thuẫn).

Vậy $L=2$, dễ có $(a_n)$ hội tụ ($lima_n=1$, xét khoảng nghiệm như trên).

(Q.E.D)




#684338 C/m: $max (n)\leq [k!e]-1$

Đã gửi bởi redfox on 13-06-2017 - 14:21 trong Tổ hợp và rời rạc

Bổ đề: $\left \lceil \frac{\left \lfloor k!e \right \rfloor}{k} \right \rceil=\left \lfloor \left ( k-1 \right )!e \right \rfloor+1$ (có thể chứng minh nhờ công thức: $e=\sum_{k=1}^{\infty }\frac{1}{k!}$).

Ta định nghĩa: $A-B=\left \{ a-b|a\in A,b\in B,a>b \right \}$. Để ý rằng $A\subset C,B\subset D\Rightarrow A-B\subset C-D$.

Ta sẽ chứng minh nếu tập $N\subset \mathbb{Z}^+$ có tập con $M$ thỏa mãn $\left | M \right |\geq \left \lfloor k!e \right \rfloor,M-M\subset N$ thì không tồn tại cách phân hoạch thỏa mãn đề. Ta chứng minh bằng quy nạp theo $k$.

Với $k=1$, dễ dàng chứng minh bài toán đúng.

Giả sử $k-1$ đúng, xét phân hoạch $N$ thành $k$ tập hợp $A_1,A_2,...,A_k$. Theo Dirichlet, tồn tại $A_i$ sao cho $\left | A_i\cap M \right |\geq \left \lceil \frac{\left | M \right |}{k} \right \rceil\geq \left \lfloor \left ( k-1 \right )!e \right \rfloor+1$. Gọi $x$ là phần tử lớn nhất của $A_i\cap M$. Đặt $M'=\left \{ x \right \}-A_i\cap M$, ta có $\left | M' \right |\geq \left \lfloor \left ( k-1 \right )!e \right \rfloor$. Dễ thấy $M',M'-M'\subset A_i\cap M-A_i\cap M\subset M-M\subset N;M',M'-M'\subset A_i-A_i$

TH1: $M'\cap A_i\neq \varnothing$ hoặc $\left ( M'-M' \right )\cap A_i\neq \varnothing$, ta chọn được ba số trong $A_i$, một số bằng tổng hai số còn lại nên cách phân hoạch này không thỏa mãn.

TH2: $M',M'-M'\subset N'=N/A_i$, áp dụng bài toán với $k-1,N',M'$ với cách phân hoạch $N'$ thành $k-1$ tập hợp $A_1,A_2,...,A_k$ (trừ $A_i$), cách phân hoạch này không thỏa mãn với $N'$ nên cách phân hoạch $N$ thành $k$ tập hợp $A_1,A_2,...,A_k$ như trên cũng không thỏa mãn đề.

Áp dụng bài toán với $N=M=\left \{ 1,2,..,n \right \},n\geq \left \lfloor k!e \right \rfloor$ ta thấy không tồn tại cách phân hoạch thỏa mãn, vậy $n\leq \left \lfloor k!e \right \rfloor-1$.

(Q.E.D)




#662092 Chứng minh rằng: $\left | S_1 \right |-\left | S_2 \...

Đã gửi bởi redfox on 15-11-2016 - 22:41 trong Tổ hợp và rời rạc

Ta chuyển bài toán về dạng tổng quát hơn: Cho tập $S$ gồm số chẵn phần tử, mỗi tập con được gán một số từ $1$ đến $2^k$ sao cho nếu $A\subset B$ thì số gán cho $B$ lớn hơn. $S_1$ là tập các tập con được gán số $a\leq x\leq b$ có số phần tử chẵn, $S_2$ định nghĩa tương tự.

Nếu hai tập con chẵn trong $S_1$ thỏa $A\subset B$ thì giữa nó có một tập con lẻ trong $S_2$, nếu ta bỏ một trong hai tập con trên và bỏ phần tử lẻ thì hiệu trên không giảm. Vậy ta chỉ cần chứng minh: Nếu tập $I$ gồm các tập con của $S$ không có tập nào là con của tập kia thì $\left | I \right |\leq C_k^{\frac{k}{2}}$.

Gọi $N_i$ là tập các tập con có $i$ phần tử. Ta sẽ chứng minh tồn tại đơn ánh $f_i$ từ $N_i$ đến $N_{i+1}$ thỏa $A\subset f(A)$với $i<\frac{k}{2}$.

Xét đồ thị lưỡng phân $G=(N_i\cup N_{i+1},E)$, nếu $A\subset B$ thì $A$ nối với $B$. Với $X\subset N_i$, số các cạnh nối với $X$ là $E=(k-i)\left | X \right |$, bậc của các phần tử của $N_{i+1}$ là $i$ nên $E$ không lớn hơn $i\left | G(X) \right |$ mà $k-i>i$ nên $G(X)\geq X$, áp dụng định lí Hall ta có điều phải chứng minh.

Tương tự tồn tại đơn ánh $f_i$ từ $N_i$ đến $N_{i-1}$ với $i>\frac{k}{2}$ và $f(A)\subset A$. Sử dụng các đơn ánh, ta đưa các tập con của $I$ về các tập con có $\frac{k}{2}$ phần tử. Vì không có tập nào là con của tập kia nên các tập con sau khi chuyển đổi phân biệt. Vậy $\left | I \right |\leq\left | N_{\frac{k}{2}} \right |=C_k^{\frac{k}{2}}$.

(Q.E.D)

Có cách nào đơn giản hơn không?




#683918 số hoán vị có tính chất P lớn hơn số hoán vị không có tính chất P(hoán vị đề...

Đã gửi bởi redfox on 10-06-2017 - 14:56 trong Tổ hợp và rời rạc

Trong trường hợp $n=1,2$ thấy có vẻ nó ko thỏa bài nhỉ (còn hơi lúng túng chỗ này).

 

Gọi $A_i$ với $i =\overline{1,n}$ là tập hợp các hoán vị thõa mãn tính chất: có cặp $i$ và $n+i$ đứng cạnh nhau trong hoán vị đó.

Dễ thấy trong một hoán vị thì số tính chất thõa mãn trên không quá $\left [  \frac{n}{2} \right ]$. Nên theo nguyên lý bù trừ có số tính chất P :

$\left |\bigcap_{i=1}^{n} A_i \right | = \sum_{I=\left \{i_1,i_2,...,i_k \right \} \\ I \subset \left \{1,2,...,n \right \}  } (-1)^{|I|+1} \left |\bigcup_{j=1}^{k} A_{i_{j}} \right |$ . Dễ dàng tính được $\left |\bigcup_{j=1}^{k} A_{i_{j}} \right | = 2^k(n-k)!\binom{2n-2k}{n-2k} $ kết hợp với tính đối xứng của các tính chất. Suy ra $\left |\bigcap_{i=1}^{n} A_i \right | = \sum_{k=1}^{\left [  \frac{n}{2} \right ]} (-1)^{k+1} 2^k(n-k)!\binom{n}{k}\binom{2n-2k}{n-2k} $

Từ đây ta sử dụng quy nạp để chứng minh nó lớn hơn $\frac{2n!}{2.n!}$. Suy ra điều phải chứng minh với $n>2$.

 

Lời giải còn hơi mơ hồ . Ai có cách khác ko

Xét ánh xạ từ các hoán vị không có tính chất $P$ đến các hoán vị có tính chất $P$: $f\left ( x_1,x_2,...,x_n \right )=\left ( x_2,...,x_{k-1},x_1,x_k,...,x_{2n} \right )$ với $x_k$ thỏa $\left | x_1-x_k \right |=n$ (dễ thấy $\left | x_{k-1}-x_1 \right |\neq n$. Giả sử $f\left ( x_1,x_2,...,x_{2n} \right )= \left ( z_1,z_2,...,z_{2n} \right )$, xét chỉ số $i>1$ thỏa $\left | z_i-z_{i+1} \right |=n$ (có một chỉ số duy nhất), ta xác định được ngay bộ $\left ( x_1,x_2,...,x_n \right)$: $x_1=z_i,k=i+1,x_l=z_l\left ( l\geq k \right ),x_l=z_{l+1}\left ( l<k-1 \right )$, vậy đây là đơn ánh. Xét hoán vị $\left ( 1,n+1,... \right )$, ta thấy nó không là ảnh của hoán vị không có tính chất $P$. Vậy đây là đơn ánh, không phải là toàn ánh. Vậy số hoán vị có tính chất $P$ nhiều hơn.

(Q.E.D)




#681402 Tưởng dễ mà khó không tưởng :p

Đã gửi bởi redfox on 21-05-2017 - 14:35 trong Số học

http://www.sciencedi...022314X02000495

không dễ thật.




#653556 Tìm k,a,b nguyên dương

Đã gửi bởi redfox on 10-09-2016 - 13:41 trong Số học

$k=5$ đúng rồi nhưng bạn mới chỉ tìm cặp $(a,b)$ sao cho $a+b$ nhỏ nhất. Mà bạn nên tìm cặp $(a;b)$ sao cho $b$ nhỏ nhất là $b=1$

Các cặp còn lại xác định bởi dãy truy hồi: $x_1=1, x_2=2; x_{n+2}=5x_{n+1}-x_n$ hoặc $y_1=1, y_2=3; y_{n+2}=5y_{n+1}-y_n$ và lấy $(a;b)=(x_{n+1};x_n)$ hoặc $(a;b)=(y_{n+1};y_n)$ và hoán vị.

Latex là \mid nhé. Chúc bạn học giỏi.




#693103 Có tồn tại hình $A$ hay không?

Đã gửi bởi redfox on 15-09-2017 - 21:16 trong Tổ hợp và rời rạc

Bài 1: Không. Ta sẽ chứng minh chu vi $A$ không nhỏ hơn $B$. Ta có thể giả sử $A$ là hình lồi (chu vi bao lồi của $A$ không lớn hơn $A$ và bao lồi của $A$ chứa $B$). Ta chia đường biên của $B$ thành $n$ cung bằng nhau bởi các điểm $B_1,...,B_n$ theo chiều kim đồng hồ, các điểm này tạo thành đa giác lồi $P$. Từ hai điểm $B_i,B_{i+1}$ kẻ hai tia vuông góc với cạnh $B_iB_{i+1}$ và nằm phía ngoài $P$ cắt $A$ tại cung $A_i$. Vì đa giác $P$ và $A$ lồi nên các cung không giao nhau hoặc giao nhau tại một điểm. Ta có chiều dài cung $A_i$ không nhỏ hơn chiều dài cạnh $B_iB_{i+1}$ (đường ngắn nhất nối hai điểm phân biệt là đoạn thẳng, $B_iB_{i+1}$ là khoảng cách giữa hai tia trong cách dựng cung $A_i$ trên). Cho $n$ đến vô cùng ta có đpcm.

Bài 2: Không. Xét cạnh nối hai điểm tùy ý trong $B$ và mặt phẳng qua hai điểm đó, cạnh đó nằm trong tiết diện của mặt phẳng đó với $B$ nên nằm trong $B$, vậy khối $B$ lồi. Sau đó chia bề mặt của $B$ bằng các mặt phẳng cách đều nhau song song với mỗi cặp hai trục tọa độ. Sau đó chiếu như CM trên.

(mình không chắc chắn lắm về lời giải.)




#650225 Tìm số nguyên dương $k$ nhỏ nhất để không có hai số nào cùng màu là...

Đã gửi bởi redfox on 18-08-2016 - 15:16 trong Tổ hợp và rời rạc

$k=\left\lfloor\log_2n\right\rfloor+1$
Xét $k$ số $1;2;2^2;...;2^{k-1}$. Hiển nhiên ta phải dùng ít nhất $k$ màu.
Giữ nguyên $\left\lfloor\frac{n}{2}\right\rfloor$ số đầu tiên của dãy, tô màu các số còn lại và tiếp tục làm vậy với các số chưa được tô.
Cách tô này dùng đúng $k$ màu. Gọi $m$ và $n$ là hai số cùng tô bởi một màu. Giả sử $m<n$. Từ cách tô dễ thấy $2m>n$. Vậy cách tô trên thỏa mãn.



#652346 Chứng minh rằng vị trí của O là không đổi sau khi một số hữu hạn bước.

Đã gửi bởi redfox on 02-09-2016 - 07:51 trong Tổ hợp và rời rạc

Gọi các tâm hình tròn theo thứ tự di chuyển là $O_1;O_2;...;O_k;...$. Gọi: 

-Số điểm nằm trong $(O_k;r)$, tổng bình phương khoảng cách từ $O_k, O_{k+1}$ đến các điểm đó lần lượt là $s_k, S_k, S'_k$.

-Số điểm nằm trong $(O_k;r)$ và nằm ngoài $(O_{k+1};r)$, tổng bình phương khoảng cách từ $O_{k+1}$ đến các điểm đó lần lượt là $a_k, A_k$.

-Số điểm nằm ngoài $(O_k;r)$ và nằm trong $(O_{k+1};r)$, tổng bình phương khoảng cách từ $O_{k+1}$ đến các điểm đó lần lượt là $b_k; B_k$.

Ta có:

-$s_k+b_k=s_{k+1}+a_k$.

-$S'_k+B_k=S_{k+1}+A_k$.

-Dựa vào vị trí tương đối của các điểm đối với $(O_{k+1};r)$, ta có $A_k\geq a_kr^2, B_k\leq b_kr^2$.

-Áp dụng Lagrange, ta dễ có $S'_k\leq S_k$.

Từ đây ta có $S_k-s_kr^2\geq S_{k+1}-s_{k+1}r^2$, dùng đơn biến ta dễ có với $k$ đủ lớn, $O_k\equiv O_{k+1}\equiv O_{k+2}\equiv ...$ hay vị trí của $O$ không đổi.

(Q.E.D)




#683908 Tính số đường đi từ $(0,0)$ đến $(p,q)$

Đã gửi bởi redfox on 10-06-2017 - 14:17 trong Tổ hợp và rời rạc

Nếu $\left ( p;q \right )=1$ thì đúng vậy.

Xét dãy các bước đi là $\vec{v}_1,\vec{v}_2,...,\vec{v}_{p+q}$ và các điểm nguyên trên đường đi trừ $\left ( p,q \right )$ là $A_1,A_2,...,A_{p+q}$.

Với đường đi bất kì và số $1< k\leq p+q$, ta hoán vị các bước đi thành $\vec{v}_k,\vec{v}_{k+1},...,\vec{v}_{p+q},\vec{v}_1,\vec{v}_2,...,\vec{v}_{k-1}$ và hoán vị tên các điểm nguyên một cách tương tự, ta được một đường đi mới. Làm như vậy ta được $p+q$ đường đi khác nhau (vì $\left ( p;q \right )=1$). Trên đường đi đó, xét các đường thẳng đi lần lượt qua các điểm $A_1,A_2,...,A_{p+q}$ và song song với $qx-py=0$. Vì $\left ( p;q \right )=1$ nên các đường thẳng đó phân biệt, xét đường thẳng nằm cao nhất (tức là đường thẳng $qx-py+k=0$ có $k$ lớn nhất) đi qua $A_i$. Ta để ý sau khi hoán vị các bước đi và tên các điểm, đường thẳng đi qua $A_i$ vẫn nằm cao nhất. Vậy trong $p+q$ đường đi đó, chỉ có một đường đi thỏa mãn đề bài là đường đi có điểm $\left ( 0,0 \right )$ có tên là $A_i$. Ta có $\frac{1}{p+q} \binom{p}{p+q}$ bộ $p+q$ đường đi nên có $\frac{1}{p+q} \binom{p}{p+q}$ đường đi thỏa mãn đề bài.

Có lời giải trong TH $\left ( p;q \right )\neq 1$ không?




#694593 chứng minh $\chi (G)\leq \Delta(G)$

Đã gửi bởi redfox on 11-10-2017 - 19:50 trong Tổ hợp và rời rạc

http://myweb.facstaf...kara/brooks.pdf brook's theorem nhé.




#684078 Chứng minh rằng $m|ak!$.

Đã gửi bởi redfox on 11-06-2017 - 14:28 trong Đa thức

Cho đa thức $P\left ( x \right )\in \mathbb{Z}\left [ x \right ]$ có bậc là $k$, hệ số cao nhất là $a$ và số nguyên dương $m$.

a) Biết rằng $m|P(x),\forall x\in \mathbb{Z}$. Chứng minh rằng $m|ak!$.

b) Cho $m|ak!$. Chứng minh tồn tại đa thức $P\left ( x \right )\in \mathbb{Z}\left [ x \right ]$ có bậc là $k$, hệ số cao nhất là $a$ thỏa mãn $m|P(x),\forall x\in \mathbb{Z}$.




#692183 $\sum_{n=2}^m a_2a_3...a_n<1$

Đã gửi bởi redfox on 03-09-2017 - 09:06 trong Số học

Định nghĩa: $\sum_{p\in \mathbb{P}},\prod_{p\in \mathbb{P}}$ là tổng, tích lấy trên tất cả các số nguyên tố.

Một số đánh giá: $\sum_{p\in \mathbb{P}}\frac{1}{p^2}<\sum_{k=4}^{\infty }\frac{1}{k^2}<\sum_{k=3}^{\infty }\frac{1}{k\left ( k+1 \right )}=\frac{1}{3}$

$\frac{n}{n-1}\leq \frac{3}{2},\forall n\geq 3$

Áp dụng AM-GM: $\prod_{k=2}^{n}a_k\leq \left ( \frac{\sum_{k=2}^{n}a_k}{n-1} \right )^{n-1}=\left ( \frac{\sum_{p\in P}\frac{\left \lfloor \frac{n}{p} \right \rfloor}{p}}{n-1} \right )^{n-1}\leq \left (\frac{n}{n-1} \sum_{p\in P}\frac{1}{p^2} \right )^{n-1}<\left ( \frac{1}{2} \right )^{n-1},\forall n\geq 3$

(chú ý rằng $\left \lfloor \frac{n}{p} \right \rfloor$ là số các số nguyên từ $2$ đến $n$ chia hết cho $p$, cũng là số lần xuất hiện của số hạng $\frac{1}{p}$

Vậy: $\sum_{n=2}^{\infty }\prod_{k=2}^{n}a_k<\frac{1}{2}+\sum_{n=3}^{\infty }\left ( \frac{1}{2} \right )^{n-1}=1$

(Q.E,D)




#692051 Có tồn tại vô hạn số tự nhiên q thỏa mãn $\left[\alpha q^2...

Đã gửi bởi redfox on 01-09-2017 - 17:25 trong Số học

Câu trả lời là có. Viết $\alpha =\left [ x_0;x_1,x_2... \right ]$ (liên phân số vô hạn). Xét dãy $p_{-2}=0,p_{-1}=1,p_n=x_np_{n-1}+p_{n-2},q_{-2}=1,q_{-1}=0,q_n=x_nq_{n-1}+q_{n-2}$. Khi đó $0<\alpha -\frac{p_{2k}}{q_{2k}}<\frac{1}{q_{2k}^2}\Rightarrow p_{2k}q_{2k}<\alpha q_{2k}^2<p_{2k}q_{2k}+1\Rightarrow \left \lfloor \alpha q_{2k}^2 \right \rfloor=p_{2k}q_{2k}$ (chứng minh chi tiết epsilon 4 p25-36)




#665431 Chứng minh tập các ước số nguyên tố của dãy vô hạn.

Đã gửi bởi redfox on 22-12-2016 - 07:40 trong Số học

Cho dãy số nguyên dương tăng ngặt $\left ( a_n \right )_{n=1}^\infty$ thỏa $a_n\leq p_n,\forall n\in \mathbb{Z}$, trong đó $p_n$ là số nguyên tố thứ $n$. Chứng minh tập các ước số nguyên tố của dãy vô hạn.




#651545 Chứng minh rằng: $(\sum_{i=1}^{n}p_i.x_i^2)-(...

Đã gửi bởi redfox on 27-08-2016 - 22:01 trong Các bài toán Đại số khác

Biểu thức trên là tam thức bậc hai đối với $x_i$. Hệ số của $x_i^2$ là $p_i-p_i^2\geq 0$ nên đạt giá trị lớn nhất tại một trong hai đầu mút $a$ hoặc $b$ nên ta chỉ xét trường hợp $x_i$ nhận hai giá trị trên. Đặt $k=\sum_{i\mid x_i=a}p_i, l=\sum_{i\mid x_i=b}p_i$với $k,l\in [0;1], k+l=1$. ta có

$\sum_{i=1}^{n}p_ix_i^2-(\sum_{i=1}^{n}p_ix_i)^2= ka^2+lb^2-(ka+lb)^2=(k+l)(ka^2+lb^2)-(ka+ly)^2=kl(b-a)^2\leq (\frac{k+l}{2})^2(b-a)^2=(\frac{b-a}{2})^2$

(Q.E.D)




#646868 Chứng minh đa thức có $n$ nghiệm thực

Đã gửi bởi redfox on 28-07-2016 - 10:33 trong Đa thức

Chứng minh rằng với mỗi số nguyên dương $n$, đa thức $\sum_{k=0}^{n}2^{k(n-k)}x^k$ có đúng $n$ nghiệm thực.




#646867 có bao nhiêu số

Đã gửi bởi redfox on 28-07-2016 - 10:27 trong Tổ hợp và rời rạc

Gọi $F_n$ là số số tự nhiên thỏa mãn đề bài.

Ta sẽ tính $F_{n+2}$

Nếu chữ số đứng thứ hai từ trái sang phải khác $0$ thì có $F_{n+1}$ cách chọn $n+1$ chữ số sau cùng và $8$ cách chọn chữ số đầu tiên. Vậy có $8F_{n+1}$ số.

Nếu chữ số đứng thứ hai bằng $0$ thì có $F_n$ cách chọn $n$ chữ số sau cùng và $9$ cách chọn chữ số đầu tiên. Vậy có $9F_n$ số.

Vậy $F_{n+2}=8F_{n+1}+9F_n$. Ta lại có $F_1=1, F_2=17$. Bằng quy nạp ta chứng minh được $F_n=(4(-1)^n+9^n)/5$.




#652998 C/m:$\forall r\in Q+$ thì có duy nhất n là số dương sao c...

Đã gửi bởi redfox on 06-09-2016 - 14:09 trong Dãy số - Giới hạn

Với $a,b\in \mathbb{Z}^+;gcd(a,b)=1$, viết $\frac{a}{b}=k_m+\frac{1}{k_{m-1}+\frac{1}{...+\frac{1}{k_2+\frac{1}{k_1}}}}$ trong đó $m,k_1,k_2,...,k_{m-1}\in \mathbb{Z}^+;k_m\in \mathbb{N}$.

Khi đó $n=1\underbrace{0..0}_{k_1-1}1\underbrace{0...0}_{k_2-1}1...1\underbrace{0...0}_{k_{m-1}-1}1\underbrace{0...0}_{k_m}$ (viết dưới dạng nhị phân) thoả mãn $x_n=\frac{a}{b}$.

Với mỗi phân số $\frac{a}{b}$, xác định duy nhất các số $m,k_1,k_2,...,k_m$ và xác định duy nhất $n$.

Với mỗi $n\in \mathbb{Z}$, xác định duy nhất các số $m,k_1,k_2,...,k_m$ và xác định duy nhất phân số $\frac{a}{b}$

Vậy đây là song ánh, dễ suy ra bài toán.

(Q.E.D)




#656052 max $F(n)$

Đã gửi bởi redfox on 29-09-2016 - 22:29 trong Tổ hợp và rời rạc

$k=1$ thì $max F(n)=1$. Ta xét $k>1$.

Chọn $n=k!$.

Ta có $\binom{n}{k}=\prod_{i=1}^{k-1}(n-i)$. Vậy $m\in A,\forall m\in \mathbb{N},n-k+1\leq m\leq n-1$. Vậy $max F(n)\geq k-1$

Gọi $p$ là ước nguyên tố của $k$. Ta đặt $a=max v_p(n-i)$, chọn $m$ thoả mãn. Ta đặt $n-m=xp^a$

Ta có $v_p(\binom{n}{k})=\sum_{i=1}^{\infty }(\left \lfloor \frac{n}{p^i} \right \rfloor-\left \lfloor \frac{n-k}{p^i} \right \rfloor-\left \lfloor \frac{k}{p^i} \right \rfloor)$.

-$p^a>p$:

Với $i>a$, ta có $\left \lfloor \frac{n}{p^i} \right \rfloor\geq \left \lfloor \frac{n-k}{p^i} \right \rfloor\geq \left \lfloor \frac{x-1}{p^{i-a}} \right \rfloor=\left \lfloor \frac{x}{p^{i-a}} \right \rfloor,\left \lfloor \frac{n}{p^i} \right \rfloor=\left \lfloor \frac{xp^a+m}{p^i} \right \rfloor=\left \lfloor \frac{x}{p^{i-a}} \right \rfloor$( $x$ không chia hết cho $p$). Từ đó các số hạng với $i=1,i>a$ bằng $0$, các số hạng còn lại không lớn hơn $1$ suy ra $v_p(\binom{n}{k})< a-1= v_p(n-m)$ hay có ít nhất một số $n-i$ nào đó không thuộc $A$. Vậy $F(n)\leq k-1$.

Từ trên ta có $max F(n)=k-1$.

-$p^a\leq k$:

Tương tự ta có $max F(n)=k-1$.




#670423 Tồn tại một số nguyên tố cùng nhau với $n-1$ số còn lại.

Đã gửi bởi redfox on 30-01-2017 - 08:39 trong Số học

Chứng minh rằng trong $n$ ($n\geq 2$) số nguyên dương liên tiếp, tồn tại một số nguyên tố cùng nhau với $n-1$ số còn lại.




#646643 $2-\frac{1}{n}< \alpha < 2$

Đã gửi bởi redfox on 26-07-2016 - 22:05 trong Số học

Ta có $x=1$ không phải nghiệm của phương trình. Nhân hai vế phương trình với $x-1$ ta được phương trình $(2-x)x^n=1$.

Ta có $(2-x)x^n=1>0\Rightarrow 2-x>0\Rightarrow x<2$

Nếu $0<x<1$ thì $(2-x)x^n=x(2-x)x^{n-1}<(2-x+x)^2/4=1$. Vậy $x>1$. Đặt $y=x-1$ ta được phương trình $(1-y)(1+y)^n=1$ với $y>0$

Áp dụng BĐT Bernoulli (dấu $=$ không xảy ra) ta có $1=(1-y)(1+y)^n>(1-y)(1+ny)=1+(n-1)y-ny^2\Rightarrow ny^2>(n-1)y\Rightarrow y>1-1/n\Rightarrow x>2-1/n$. Vậy $2-1/n< \alpha < 2$.

(Q.E.D)




#656201 Chứng minh $B, C, E$ thẳng hàng.

Đã gửi bởi redfox on 01-10-2016 - 10:23 trong Hình học

Cho tam giác $ABC$, tâm đường tròn nội tiếp $I$. Điểm $D$ thỏa mãn tứ giác $BICD$ là tứ giác điều hòa. Kẻ đường kính $IE$ của đường tròn ngoại tiếp tam giác $ADI$. Chứng minh $B, C, E$ thẳng hàng.




#670684 USA December TST for the IMO 2017

Đã gửi bởi redfox on 07-02-2017 - 23:05 trong Thi HSG Quốc gia và Quốc tế

bài 1:

$g(n;t)=\left \lceil \frac{n}{t} \right \rceil$, ví dụ ta cho $\left \lceil \frac{n}{t} \right \rceil-1$ đội, mỗi đội mặc $t$ màu áo phân biệt, đội còn lại có không quá $t$ màu áo.

Đầu tiên ta chọn đội bất kì có $x_1$ màu áo. Nếu màu áo chưa đủ $n$ ta chọn đội có $x_2$ màu áo còn lại. Chọn như vậy đến khi đủ $n$ màu áo và $k$ đội "có màu sắc nhận dạng được. Dễ thấy $x_i \leq t$ Ta có $n=\sum_{i=1}^{k} x_i\leq kt\Rightarrow k\geq \left \lceil \frac{n}{t} \right \rceil$.

(Q.E.D)